LSAT and Law School Admissions Forum

Get expert LSAT preparation and law school admissions advice from PowerScore Test Preparation.

 Administrator
PowerScore Staff
  • PowerScore Staff
  • Posts: 8919
  • Joined: Feb 02, 2011
|
#40383
Complete Question Explanation
(The complete setup for this game can be found here: lsat/viewtopic.php?t=13080)

The correct answer choice is (A)

This question represents a direct test of your Not Laws, many of which can only be inferred following a Templates-based approach. Without the use of templates, the only alternative is a plug-and-chug approach, in which we create local diagrams for each answer choice in the hopes of eliminating those that cannot be true. Clearly, this would be a far less efficient approach than the use of templates.

Answer choice (A) is the correct answer choice, as shown in Template 2.2 in the setup.

Answer choice (B) is incorrect, because there is no template in which Q is fifth. Without the use of templates, you need to examine whether or not Q can be fifth: If Q were fifth, this would force T to be first (second rule), and R to be second (first rule). Since either Q or V must be third (third rule), V would have to be third, leaving S to be fourth:
PT72_Game_#2_#10_diagram 1.png
In this diagram, S and Q would be adjacent, violating the last rule of the game.

Answer choice (C) is incorrect, because there is no template in which V is first. If V were first, then R would be second (first rule), Q would be third (third rule), and T would be fifth (second rule):
PT72_Game_#2_#10_diagram 2.png
Once again, S and Q are forced to be adjacent, violating the last rule of the game.

Answer choice (D) is incorrect, because there is no template in which V is second. If V were second, then R would be first (first rule), Q would be third (third rule), and T would be fifth (second rule):
PT72_Game_#2_#10_diagram 3.png
As with answer choice (C), this would force S and Q to be adjacent, in violation of the last rule.

Answer choice (E) is incorrect, because there is no template in which V is fifth. If V were fifth, then T would be first (second rule), R would be second (first rule), and Q would be third (third rule):
PT72_Game_#2_#10_diagram 4.png
As with answer choices (B), (C), and (D), this would force S and Q to be adjacent, in violation of the last rule.

Get the most out of your LSAT Prep Plus subscription.

Analyze and track your performance with our Testing and Analytics Package.